Ryan invested $5000 in an account that averaged 5% growth per year. Which equation would be used to best model the situation?

Answers

Answer 1

Answer:

f(x)=5000(1+0.05)^x

Step-by-step explanation:

Use the formula: f(x)=P(1+b)^x

P is principal or y-intercept. 5000 is principal.

b is the percentage in decimal form. 0.05 is b.

x is time, in this case years. Since there is not a certain number of years leave x as x.

f(x)=5000(1+0.05)^x

Hope this helps!

If not, I am sorry.


Related Questions

I'm having trouble with inequalities, any help will be appreciated, thanks.

Answers

Answer:

-3/2 ≤ x < 5

Decimal form:

-1.5 ≤ x < 5

(hope this helps)

You can walk over the mountains to
your campsite or take a safer, easier trip around the mountains. You
are on the ground at P(0, 0) and the campsite is at C(-2, 7). You
travel around the mountain by going to A(2, 5). The coordinate
system is measured in meters. Draw a diagram of the situation.
Find PA and AC.

Answers

Answer:

PA = [tex]\sqrt{29}[/tex]

AC =[tex]\sqrt[2]{5}[/tex]

Step-by-step explanation:

distance formula : if [tex](x_{1},y_{1}) and (x_{2},y_{2})[/tex] are two points on a line segment then distance between both the points is given by distance formula.

[tex]d = \sqrt{[(x_{2} - x_{1} )^2 +(y_{2} - y_{1})^2]}[/tex]

therefore ,

given points are , A(2, 5),  P(0, 0), C(-2, 7).

so PA = [tex]\sqrt{[({2} - {0} )^2 +({5} - {0})^2]} = \sqrt{({2} )^2 +({5} )^2}[/tex]

    PA = [tex]\sqrt{\229}[/tex]

similarly,

AC = [tex]\sqrt{[({-2} - {2} )^2 +({7} - {5})^2]} = \sqrt{({4} )^2 +({2} )^2}[/tex]

AC = [tex]\sqrt{20} = \sqrt[2]{5}[/tex]

more questions on lines and distance formula at

https://brainly.com/question/25841655

#SPJ10

write the faction in the simplest form 28/36

Answers

Answer:

7/9

Step-by-step explanation:

Divide the numerator and denominator by 4.

28/36 = 7/9

Answer:

7/9

Step-by-step explanation:

28 and 36 are both divisible by 4.

28÷4 is 7 and

36÷4 is 9

28/36 = 7/9

Solve for x.

log x = 3

Answers

-------------------------------------------------------------------------------------------------------------

Answer:  [tex]\textsf{x = 1000}[/tex]

-------------------------------------------------------------------------------------------------------------

Given: [tex]\textsf{log(x) = 3}[/tex]

Find: [tex]\textsf{The value of x}[/tex]

Solution:  In order to solve for x we need to get rid of the log which means that we need to put it to the power of the same number as the base.  When a base isn't displayed it defaults to 10 so we raise everything to the base of 10.

Get rid of log

[tex]10^{\textsf{log}_{10}\textsf{(x)}}} = 10^{\textsf{3}}[/tex][tex]x = 10^{\textsf{3}}[/tex][tex]x = 1000[/tex]

Therefore, the final answer would be that x is equal to 1000.

edg vector operations, any help appreciated!

Answers

[tex]\quad \huge \quad \quad \boxed{ \tt \:Answer }[/tex]

[tex]\qquad \tt \rightarrow \: Add \:\: -6 \hat i - 6\hat j \:\:with \:\; Vector \:\; c[/tex]

____________________________________

[tex] \large \tt Solution \: : [/tex]

Vector d can be represented as :

[tex]\qquad \tt \rightarrow \: - 2 \hat i - 2 \hat j[/tex]

Vector c can be represented as :

[tex]\qquad \tt \rightarrow \: 4 \hat i + 4\hat j[/tex]

we have to create vector d from vector c

So, let's assume a vector x, such that sum of vector x and vector c equals to vector d

[tex]\qquad \tt \rightarrow \: x + ( 4 \hat i + 4 \hat j) = - 2 \hat i - 2 \hat j[/tex]

[tex]\qquad \tt \rightarrow \: x = - ( 4 \hat i + 4 \hat j) - 2 \hat i - 2 \hat j[/tex]

[tex]\qquad \tt \rightarrow \: x = (- 4 \hat i - 2 \hat i) + ( - 4 \hat j - 2 \hat j)[/tex]

[tex]\qquad \tt \rightarrow \: x = - 6 \hat i -6 \hat j[/tex]

Henceforth, in order to get vector d, we need to add (-6i - 6j) in vector c

Answered by : ❝ AǫᴜᴀWɪᴢ ❞

Can You Please Help Me, I Would Really Appreciate It!!

Answers

Let's take this problem step by step:

To find the ordered pair of the system:

 ⇒ must set both equations equal to each other

     ⇒ solve for 'x'

Let's solve:

 [tex]x^2-2x+3=-2x+12\\x^2-2x+2x+3-12\\x^2-9=0\\(x+3)(x-3)=0[/tex]

Let's find the x-values:

 [tex](x+3)=0\\x=-3\\\\(x-3)=0\\x+3[/tex]

Let's find f(x)'s value for each 'x':

 [tex]x=-3\\f(-3)=-2(-3)+12=6+12=18\\\\x=3\\f(3)=-2(3)+12=-6+12=6[/tex]

Answer: (3, 6), (-3,18)

Hope that helps!

#LearnwithBrainly

simplify (3+√5)(3+√2)

Answers

When we simplify (3 + √5)(3 + √2), the result obtained is:

9 + 3√2 + 3√5 + √10

Surd operation

a√b × c√d = (a × c)√(b × d)

How to simplify (3 + √5)(3 + √2)

(3 + √5)(3 + √2)

Expand by clearing the bracket

3(3 + √2) + √5(3 + √2)

9 + 3√2 + 3√5 + √10

Thus,

(3 + √5)(3 + √2) = 9 + 3√2 + 3√5 + √10

Learn more about surd:

https://brainly.com/question/24700530

#SPJ1

Convert the angle from radians to degrees: [tex]\frac{5}{6}\pi[/tex] with working

Answers

150 degrees according to the unit circle.

The length of a rectangle is 7 inches and the width is 42 inches. What is the ratio,
using whole numbers, of the length to the width?

Answers

Answer:

6 : 1

Step-by-step explanation:

The length is the longer side compared to the width.

Hence, the length is 42 inches and width is 7 inches.

Taking the ratio :

Length : Width42 : 77 x 6 : 76 : 1

Answer:

1 : 6

Step-by-step explanation:

42 /7 = 6

We will have to simply the ratios in order to get the answer.

Which expression has a value of 16 when n = 5?
StartFraction 25 Over n EndFraction + 7
30 minus 3 n
7 + StartFraction 45 Over n EndFraction
n cubed minus 114

Answers

Answer:

7+45/n or 5

Step-by-step explanation:

45 divided by n or technically 5 is 9 plus 7 is 16 :)

Answer:7+45/n or 5

Step-by-step explanation:

Someone please help with this!!

Answers

Answer:

A

Step-by-step explanation:

Comment (and answer)

Both start with the plus x axis as one of the arms. The other arm of 110o goes anti clockwise until it is 30 degrees into the second quadrant.

The answer can be found by using 110 - 360 = negative angle.

So 110 - 360 = - 250 which goes clockwise from the plus x axis. The answer must be A.

Make sure you understand the concept of clockwise and anti clockwise. If you don't know, ask your teacher, write it down, and try not to forget it. The distinction comes up a lot.

Is in between 2 number on the number line

Answers

Answer:

3 and 4

Step-by-step explanation:

well you can approximate by squaring values

2^2 = 4

3^2 = 9

4^2 = 16

the value should fall somewhere between 3 and 4 since the square root of 9 is 3 and the square root of 16 is 4

Use the drawing tool(s) to form the correct answer on the provided graph.
Graph the solution to the following linear inequality in the coordinate plane.
5x - y > -3
27
Drawing Tools
Select
Line
Dashed Line
Shaded Region
Click on a tool to begin drawing.
10
8
6

Answers

The solution to the inequality 5x - y > -3 is shown in the graph.

What is an equation?

An equation is an expression that shows the relationship between two or more numbers and variables.

An inequality shows the non equal comparison of two or more variables and numbers.

The solution to the inequality 5x - y > -3 is shown in the graph.

Find out more on equation at: https://brainly.com/question/2972832

#SPJ1

Find (f • g) (x) Assume x>0

Answers

Answer:

[tex]\textsf{B.} \quad (f \cdot g)(x)=10x[/tex]

Step-by-step explanation:

Given:

[tex]\begin{cases}f(x)=\sqrt{50x}\\g(x)=\sqrt{2x}\end{cases}[/tex]

[tex]\begin{aligned}\textsf{As }(f \cdot g)(x) & = f(x) \cdot g(x)\\\implies (f \cdot g)(x)& = \sqrt{50x} \cdot \sqrt{2x}\end{aligned}[/tex]

[tex]\textsf{Apply radical rule} \quad \sqrt{a}\sqrt{b}=\sqrt{ab}:[/tex]

[tex]\begin{aligned}\implies (f \cdot g)(x) &= \sqrt{50x2x}\\& = \sqrt{100x^2}\end{aligned}[/tex]

Rewrite 100 as 10²:

[tex]\implies (f \cdot g)(x)=\sqrt{10^2x^2}[/tex]

[tex]\textsf{Apply exponent rule} \quad a^bc^b=(ac)^b:[/tex]

[tex]\implies (f \cdot g)(x)= \sqrt{(10x)^2}[/tex]

[tex]\textsf{Apply radical rule} \quad \sqrt{a^2}=a, \quad a \geq 0:[/tex]

[tex]\implies (f \cdot g)(x)=10x[/tex]

Could someone explain why the value of x is (-2).​

Answers

Answer:

x = -2

Step-by-step explanation:

2x^3 +16 = 0

We are solving for x

Subtract 16 from each side

2x^3 +16- 16 = 0-16

2x^3 = -16

Divide each side by 2

(2x^3 ) /2 = -16/2

x^3 = -8

Take the cube root of each side

[tex]\sqrt[3]{x^3} = \sqrt[3]{-8}[/tex]

x = -2

Answer:

[tex]x=-2[/tex]

Step-by-step explanation:

Given equation:

[tex]2x^3+16=0[/tex]

To solve for the unknown variable x, apply arithmetic operations to isolate the variable.

Subtract 16 from both sides:

[tex]\implies 2x^3+16-16=0-16[/tex]

[tex]\implies 2x^3=-16[/tex]

Divide both sides by 2:

[tex]\implies \dfrac{2x^3}{2}=\dfrac{-16}{2}[/tex]

[tex]\implies x^3=-8[/tex]

Take the cube root of both sides:

[tex]\implies \sqrt[3]{x^3}=\sqrt[3]{-8}[/tex]

[tex]\implies x=-2[/tex]

When taking the cube root of a negative number, the result will be negative.  To understand why, examine what happens when we cube a negative number.

When a number is cubed, it is multiplied by itself, then by itself again.

When multiplying a negative number by another negative number, the result is always positive.

When multiplying a positive number by a negative number, the result is always negative.

Therefore:

[tex]\begin{aligned}\implies (-2)^3 &= -2 \cdot -2 \cdot -2\\ & = 4 \cdot -2\\& = -8\end{aligned}[/tex]

So if -8 is cube rooted, the result is -2.

Which of the following are solutions to the equation below?
Check all that apply.
(3x - 5)^2 = 19

Answers

Answer:

To solve the answer, you can do it in 2 ways:

1) you could take the square root on both sides and solve for x

OR

2)you could multiply 3x-5 times itself and subtract 19 on both sides and use the grouping or quadratic formula method to find x.

Hope it helps.

Take the square root of each side of the equation to set up the solution for

Remove the perfect root factor

under the radical to solve

The complete solution is the result of both the positive and negative portions of the solution.

The result can be shown in multiple forms.

Fill in the table with the appropriate value to describe the amount of hot dogs eaten at a hot dog eating contest.

Which value should be placed in the empty space in the table?

A. 1/6
B. 1/4
C. 1/12
D. 1/8

Answers

20 hot dogs was eaten by the 10 people who attended the hot dog eating contest.

What is an equation?

An equation is an expression that shows the relationship between two or more numbers and variables.

Let us assume that there were 10 people at the hot dog eating contest. If each person eats 2 hot dog, hence:

Number of hot dogs eaten = 2 hot dog per person * 10 people = 20 hot dogs.

20 hot dogs was eaten by the 10 people who attended the hot dog eating contest.

Find out more on equation at: https://brainly.com/question/2972832

#SPJ1

Marcela was assigned some math problems for
homework. She answered half of them in study hall.
After school she completed seven more. If she still
has 11 problems to do, how many problems were
assigned?

Answers

Answer:

36 problems

Step-by-step explanation:

so let's say x = how many problems Marcela was assigned

she completed half of them so the amount of problems left is x/2

She then completed 7 more so now we can subtract 7. We then set this to 11 to solve for x

x/2 - 7 = 11

x/2 = 18

x = 36

2. The smallest number from the given numbers 6895, 4875 and 5689 will have its smallest digits at place. (A) ​

Answers

The smallest digit of the smallest number is at the thousands place.

What is Number System ?

The system of representing numbers is called Number System.

The given numbers are 6895 , 4875 , 5689

The smallest number among these is 4875

The smallest digit in this number is 4

4 is at thousands place

Therefore , The smallest digit of the smallest number is at the thousands place.

To know more about Number System

https://brainly.com/question/22046046

#SPJ1

The steps to simplify the expression -100 ÷ 3 x (-0.6) are shown.

Answers

Answer:

That is the answer for you step 2

Gerald purchased a rectangular plot of land. the length of the plot is 20 feet more than the width. the cost of the land was $12 per square foot. gerald also had a fence put around the entire perimeter of the plot, at a cost of s8 per linear foot. the total amount he spent on both the land and the fence was $10,560. part a write an equation in two variables for the perimeter, p, and an equation in two variables for the area, a, of the plot of land where x is the width of the plot in feet. provide evidence to support your equations

Answers

Answer:

P =2(x+ y) [tex]ft[/tex].

A =xy [tex]ft^{2}[/tex]

Step-by-step explanation:

perimeter of rectangle (P)= 2(L+B

Area of rectangle (A)        =    L×B

Given,

width = 'x'

length = y = x+20

therefore using above formula

P = 2(x+20 + x) = 2(2x+20)

A = (x+20)(x)

cost of land per square foot = $12

cost of land per linear foot   = $8

total cost of land                   = $10560

now,

P×8 + A×12   = 10560

2(2x+20)×8 +  (x+20)(x)×12 = 10560

32x + 320 + 12x²+ 240x     = 10560

12x² + 272x                         = 10240

12x² + 272x - 10240           = 0

solving this quadratic equation using Discriminant method

x = [-b ±√(b² - 4ac)]/2

where b is the coefficient of x

a is the coefficient of x²

and c is the constant term

by replacing values in the above formula

x = 19.5  , x = -42.24

as the area cannot be negative therefore -42.24 is neglected

hence x = 19.5ft

more on quadratic equations on

https://brainly.com/question/2925460

#SPJ10

The x-coordinate of the point (50, 55) is ___________.

Answers

Answer: 50

In an Ordered Pair, the first value is the x-coordinate or the abscissa and the second value is the y-coordinate or the ordinate. In the pair (50, 55), 50 is the abscissa and 55 is the ordinate.

The x-coordinate of the point (50, 55) is 50

What is graph?

A graph can be defined as a pictorial representation or a diagram that represents data or values.

The distance of point (50, 55) perpendicular to y-axis

Thus, perpendicular distance = 50 units

The x-coordinate of the point (50, 55) is 50

Learn more about graph

brainly.com/question/16608196

#SPJ2

\In a mouse population, some mice have thicker fur than others, but there are more mice with thinner fur. The climate in which these mice live has slowly gotten colder.

How will this change in the environment most likely affect the mouse population?

The adaptation of thinner fur will be selected over thicker fur, and the population will remain unchanged.
The adaptation of thicker fur will be selected over thinner fur, and the population will remain unchanged.
The adaptation of thinner fur will be selected over thicker fur, and the population will evolve.
The adaptation of thicker fur will be selected over thinner fur, and the population will evolve.

Answers

The adjustment of thicker fur will be selected over thinner fur, and the population will develop. Then the correct option is D.

What is decision-making?

Determining the proper option, acquiring evidence, and exploring various options are all steps in the decision-making process.

In a mouse population, some mice have thicker fur than others, but there are more mice with thinner fur.

The climate in which these mice live has slowly gotten colder.

The change in the environment most likely affect the mouse population will be

The adjustment of thicker fur will be selected over thinner fur, and the population will develop.

Then the correct option is D.

More about the decision-making link is given below.

https://brainly.com/question/3369578

#SPJ1

A. Yes, the points shown on the line would be part of y=0.5x
B. Yes, all proportions can be shown on a graph of this line
C. No, the points shown would not be part of y=0.5x
D. No, the proportions cannot be represented on a graph

Answers

Answer:

B

Step-by-step explanation:

given the graph of y = 0.5x

Then any point on the line satisfies y = 0.5x

consider the points given

x = 1 then y = 0.5 × 1 = 0.5 ⇒ (1, 0.5 ) is on the line

x = 4 then y = 0.5 × 4 = 2 ⇒ (4, 2 ) is on the line

What is the solution of the inequality sh
below?
9+b≤2

Answers

Answer:

b≤-7

Step-by-step explanation:

b+9≤2

Subtract 9 from both sides:

b+9-9≤2-9b≤-7

9+b≤2

subtract 9 on each side which will be -7

the answer will be b≤-7

Camille bought 120 doughnuts for $60. Her profit was $48 once she sold 80 doughnuts. Write an
equation that represents Camille's profit P, as a function of the number of doughnuts sold, n.
PLEASE HELP WITH SHOWED WORK!!!

Answers

The equation that represents Camille's profit p as a function of the number of doughnuts sold, n is P = 1.35n - 60

How to represent an equation ?

She bought 120 doughnuts for $60.

Therefore,

cost  = $60

Her profit was $48 once she sold 80 doughnuts.

p = profit

n = number of doughnuts sold.

Therefore,

let

x = amount she is selling each doughnuts

p = nx - 60

Using the information given we have:

48 = (80x) - 60

Therefore,

x = $1.35

Hence, for any number of doughnuts then, we have the profit equation as follows:

P = 1.35n - 60

learn more equation here: https://brainly.com/question/21691697

#SPJ1

Need help urgently please

Answers

1/2x+6(x+15)=12
1/2x+6x+90=12
6.5x+90=12
6 1/2x= -78
x= -12

y= -12+15
y=3
nah as smalala she hiss amal and skids is sus sike

area = ft
help me please ty:)​

Answers

find the length using the Pythagorean theorem.

Length = sqrt(20^2 - 12^2)

Length = 16 feet


area = 12 x 16 = 192 square feet

Two gymnasts are running toward each other in a floor routine, and they plan to precisely time a flip to stay synchronized for the audience. The path of the gymnasts is parabolic and modeled by the following equations, where y is the height of the flip and x is the time in seconds:

Answers

Answer:

4 secs

Step-by-step explanation:

Since they have to be synchronized, their parabolic equations must be equal to one another

3(t^2 + 9 - 6t) - (t^2 + 25 - 10t) - t + 2 = 0

2t^2 - 9t + 4

(2t - 1)(t - 4) = 0

t = 1/2, 4

From the options, the answer is 4

The gymnasts will be at the same height during their flips at two different times: 1/2 seconds and 4 seconds.

so, correct option is: C.

Here, we have,

To determine when the gymnasts will be at the same height during their flips, we need to find the time (x) at which the equations for y are equal.

The given equations are:

y = –(x – 5)² + 3

y = –3(x – 3)² + x + 1

Setting the two equations equal, we have:

–(x – 5)² + 3 = –3(x – 3)² + x + 1

Expanding the squared terms:

–(x² – 10x + 25) + 3 = –3(x² – 6x + 9) + x + 1

Simplifying the equation:

–x² + 10x – 25 + 3 = –3x² + 18x – 27 + x + 1

Combining like terms:

–x² + 10x – 22 = –3x² + 19x – 26

Rearranging the equation:

2x² - 9x + 4 = 0

To solve this quadratic equation, we can factor it:

(2x - 1)(x - 4) = 0

Setting each factor equal to zero:

2x - 1 = 0 or x - 4 = 0

Solving for x:

2x = 1 or x = 4

Dividing by 2:

x = 1/2 or x = 4

Therefore, the gymnasts will be at the same height during their flips at two different times: 1/2 seconds and 4 seconds.

To learn more on equation click:

https://brainly.com/question/14468218

#SPJ2

What expression below is equivalent to -71n -
29?
A) -29 - 71n
B) 71n + (-29)
C) 29 + 71n
D) -29 - (-71n)

Answers

Answer:

A

Step-by-step explanation:

both numbers are negative, you can rearrange them in any order provided you keep the sign

Answer: Choice A) -29 - 71n

Reason:

An expression in the form -a-b is the same as -a + (-b)

Recall we can add two numbers in any order we want. For instance, 2+3 = 3+2 since both sides are equal to 5. This is the commutative property of addition.

Therefore, the -a + (-b) is the same as -b + (-a) which in turn becomes -b - a

In short:  -a - b = -b - a

This rule lets us say that -71n - 29 = -29 - 71n

Other Questions
The nth term of a sequence is given by 3n What is the position of the term in the sequence that is the first one with a value greater than 1000? What is the approximate value of x in the diagram below? (Hint: You will need to use one of the trigonometric ratios given in the table.) sin 40 cos 40 50 0.643 0.766 0.839 tan 40 90 Under what conditions was Germany reunified? In which century did Marco Polo leave Italy Every employee works 7 hours per day. Every employee works 5 days per week. Every employee works 49 weeks per year. how hours do the employee work per year Please I need help fast. Assignment is due soon.1) A ball is thrown from the top of a building with an initial speed of 8.0 m/s at an angle of 35 above the horizontal. The building is 18 m tall. a) How long is the ball in the air? b) How far from the building does the ball land? c) What is its impact speed?2) A 65-kg person driving a car hits the gas, accelerating the car at a rate of 3.9 m/s^2. Find the magnitude and direction of the force exerted by the seat on the person's body. Remember to include both the horizontal and the vertical components. Please help! I'm not understanding this question. Giving 75 points! Please provide an explanation as to why that is the answer as well. Thank you!What is i* if i = 0?A. 0B. 6C. 8D. 9E. none of these Match each symbol on the left with its figurative meaning on the right.a white Blaga flying dovebroken chainsa red heartXXXpeace and lovesurrender and humilityfreedom and independencelove and compassion What is true about anglemsr? it must be acute. it must be a right angle. it must be equal to anglemrh. it must be equal to anglerms. Night driving is a special type of hazardous driving. to help your night vision when approaching an oncoming vehicle, you should direct your eyes to the ________ of the road. group of answer choices right side left side Write the Spanish conjugated 1 How did mining change over time? A Individual miners working independent claims took over claims abandoned by large corporations. B Large mines owned by corporations took over independent claims worked by individual miners. CAs barbed wire became available. mines were pushed into smaller and smaller areas until they went out of business. D Mining became more profitable after Native Americans were restricted to living on reservation what measuring basis where for dividing nepal into a federal state?write in five points Which describes how the parent function, f(x) = |x|, is transformed to show the functionf(x) = 0.1|x 3|?It is wider and shifted 3 units to the left.It is wider and shifted 3 units to the right.It is narrower and shifted 3 units to the left.It is narrower and shifted 3 units to the right. Jamaal knows that it is certain that he will win the election because he is the only person who is running for class treasurer. Which value represents the probability that he will win the election? A cab company charges a $5 boarding rate in addition to its meter which is $3for every mile. What is the equation of the line that represents this cabcompany's total charge?Reco Two numbers are in the ratio of 2 to 3. Their sum is 30. What are the numbers? 3. Two numbers are in the ratio of 2 to 3. Their sum is 30. What are the numbers ? PLEASE HELP ASAP!!!Rewrite the sentence to correct the error--either a misplaced modifier, a squinting modifier, a dangling modifier, or an instance of faulty parallelism.(I know that the sentence is an SM a squinting modifier I just need help with rewriting the sentence.)The general told his orderly before the battle he could go on leave. pls help1. when a magnet spins within a coil of wires, what is produced?-blue sparks-glowing light-nuclear energy-an electric current2.when an electric current flows through a wire:-a gravitational field is created around the wire-the wire becomes very cold-the wire begins to spin-a magnetic field is created around the wirehelp pls PLSS HELP ITS ANGLESSS